APWH Practice MCQ

Réussis tes devoirs et examens dès maintenant avec Quizwiz!

Based on the chart, which of the following best describes a pattern in the spread of railroads in Europe from 1830 to 1850 ?

Areas in northwestern Europe adopted railroads more extensively than areas in other parts of Europe.The table shows that the northwestern European countries of Great Britain, France, and the German states adopted railroads more extensively than did other areas of Europe.

Which of the following developments in the period circa 1250-1450 represents a situation that is the opposite of nomadic conquerors assimilating into the cultures of the conquered societies?

Conquered peoples being drawn into their conquerors' economic and cultural orbits, as illustrated by Turkic peoples converting to Islam and integrating into Muslim societies. In the tenth and eleventh centuries, Turkic peoples began converting en masse to Islam and migrating to South Asia and the Middle East. Turkic peoples such as the Seljuqs integrated into Islamic societies and established their own states.

The author uses all of the following as evidence to support his argument about the impact of smallpox on Native American populations EXCEPT

the English settlers tried to help the Native Americans who were afflicted with smallpox. While Bradford mentions that some English took pity on the Native Americans and tried to help them, this fact does not provide evidence about the effects of smallpox on Native American populations. Since Bradford does not use this statement as evidence to support his argument, it is the correct answer to this question.

Although ideas of European superiority over non-Europeans had existed for centuries, views such as those expressed in the passage provide evidence of an important change in the late nineteenth century that is best explained by the fact that they

combined old cultural prejudice with supposedly "scientific" new ideas of the biological and evolutionary basis of human differences. The attitudes evident in the caption illustrate a transition from old prejudices that emphasized the religious and cultural differences between Europeans and African, Asian, and Native American populations to pseudoscientific explanations that emphasized supposed differences in skin color, mental acuity, and other biological measures of distinction.

The "long stays in harbors" mentioned in the description of the sea journeys in the first paragraph were most likely necessary because Indian Ocean maritime trade in the period circa 1200-1450

had to take into account environmental factors such as the timing and direction of the monsoons. The passage mentions that the merchants had to stay in the harbors to wait for the winds to shift. Because the monsoon winds were seasonal and tended to flow only in one direction, merchants would often have to wait in harbors for long periods of time, depending on the direction in which they were intending to travel.

The class relationships in Manchester described by Parkinson are best explained in the context of the

continuous migration of new workers to industrial towns and the high job turnover. One of the many consequences of industrialization in Britain included the constant migration of workers to industrial cities and high job turnover, in large part because of the difficult and dangerous working conditions in the factories. Constant migration and high job turnover led to an inconsistent and varied workforce, which contributed to the lack of communication between the social classes of industrial cities such as Manchester that Parkinson describes in the paper.

Which of the following processes most immediately led to the global expansion of European political power in the period circa 1500 ?

Advances in shipbuilding and navigational methods. European advances in navigation and shipbuilding, made possible through inventions such as the astrolabe, the lateen sail, and the caravel, allowed Europeans to explore and conquer new territories, thereby expanding their global political power.

All of the following statements are factually accurate. Which could be most directly used to modify Schama's argument that the war provided "overwhelming evidence of a new-found British social cohesiveness and mutual loyalty"?

After a brief period of supporting the war effort, the Indian National Congress continued its anti-imperialist agitation, with Gandhi launching a Quit India campaign in 1942.Schama asserts that the British had established a unified nation across all social classes. Yet many people in Britain's colony of India did not sustain their loyalty to the British cause. Gandhi and his followers pursued a different goal, namely, Indian independence through the Quit India campaign, rather than aiding the British state in defeating the Nazis as Schema claims. Because this development shows that the indigenous populations in Britain's colonies did not fully support the British war effort, it would modify Schama's argument.

Considering global trends in late-nineteenth-century migration movements, the specific migrations that produced the demographic situation shown in the table most likely contributed to which of the following social or political changes in India?

An increase in the number of Indian women engaging in occupations that had formerly been held by men. As a result of the migration process, some women in India engaged in occupations that were formerly held by men, because the indentured workers who migrated to the Caribbean tended to be male.

The instructions to Lugard in articles (d) and (f) best reflect which of the following regarding the purpose of the board of directors' letter?

As a commercial enterprise, the British Royal Niger Company hoped that Lugard's voyage would allow it to expand its exports of African raw materials to Europe and its imports of European finished goods to Africa.Article (d) orders Lugard to record the existence of precious metals such as gold, and article (f) instructs Lugard to convey to African chiefs how economically advantageous it would be for them to allow European merchants to import goods from Europe to the chiefs' territories. This shows that the British Royal Niger Company was interested in obtaining African raw materials and exporting European finished goods to Africa.

Which of the following best explains why the passage is likely NOT a reliable source of information regarding the level of popular support and enthusiasm for the Chinese government's push for iron and steel in 1958?

As a government official, the author would have felt compelled to overstate the success of communist policies of resource redistribution to demonstrate his loyalty to the party.The fact that the author was a Chinese communist official writing in a government-approved foreign-language publication strongly suggests that the picture of the industrialization reforms painted in the pamphlet would be heavily biased in favor of the Chinese government's official position and would not reflect true levels of popular support for the government's policies.

Which of the following could be most effectively used to challenge the objectivity of Vandana Shiva's arguments in her book The Violence of the Green Revolution?

As an environmentalist and anti-globalization activist, Shiva may have been selective in presenting the scientific data regarding the effects of the Green Revolution. The author's political positions as an environmentalist and anti-globalization activist can be legitimately seen as a source of potential bias in her presentation of the data. The language Shiva uses in the passage also suggests a tone of advocacy, rather than objectivity (for example, asserting that agribusiness technologies cannot but destroy "the very basis of agricultural production").

The relationship between levels of military personnel and state revenues as shown in the table is best understood in the context of which of the following global developments in the period 1450-1750 ?

As military forces expanded and became more professionalized, states were forced to develop new ways to generate revenue. The data in the table show that as military forces increased in number, state revenue expanded as well. The expansion of military personnel was largely the result of the establishment of standing armies, in which soldiers were paid a regular wage to serve in the state's military. To pay for ever-expanding armies, states needed to collect more and more revenue.

The type of weaponry used in the siege, as described in the passage, could also help to explain the expansion of all of the following states in the period 1450-1750 EXCEPT the

Aztec (Mexica) Empire. While the Aztec (Mexica) Empire did expand its territory through conquest in the fifteenth and early sixteenth centuries, it did not use gunpowder weapons during its conquests.

Compared with late-nineteenth-century East or South Asian migrants to other world regions, the ancestors of the populations shown in the table likely received a somewhat better reception in their host societies for all of the following reasons EXCEPT:

Because of the ethnic and religious diversity of Caribbean colonial societies, there was no prejudice against newcomers, and immigrants, including Indian laborers, were generally welcomed by local populations.Prejudice against newcomers was a prominent feature of Caribbean colonial societies despite the fact that they were ethnically and religiously diverse. This option is, therefore, factually inaccurate and the correct answer to the question.

Based on the information in the table, the Black Death outbreak of the bubonic plague struck Essex County in which of the following periods?

Between 1320 and 1350. According to the table, the population declines most severely between 1320 and 1350, indicating that the Black Death struck during this period.

Which of the following best explains the similarity between Hutu views of the Tutsi in the "Hutu Ten Commandments" and the treatment of Jews in Nazi Germany?

Both the Tutsi and Jews were persecuted based on ethnic prejudice.Jews in Nazi Germany and the Tutsi in Rwanda were both persecuted because the majority populations in both states believed that ethnic differences made those populations inferior, dangerous, and untrustworthy.

Which of the following best describes the comparative change in kilometers of railroad lines in Britain and France from 1830 to 1850 ?

Britain possessed more railway lines in 1830 and expanded its lead significantly over France by 1850. Britain increased its railway lines from 157 kilometers to 9,787 kilometers, while France increased its railway lines from 31 kilometers to 2,915 kilometers.

Which of the following best explains Niebuhr's likely purpose in recounting Soviet actions in postwar Europe?

By pointing out the hypocrisy of Soviet policies, he hoped to encourage a more confrontational stance on the part of the United States government.Niebuhr was concerned that the United States would withdraw from European politics following the Second World War as it had after the First World War. Moreover, he was concerned that both the United States and Europe would attempt to appease Joseph Stalin as they had Adolf Hitler during the early 1930s. Therefore, by pointing out aggressive and antidemocratic Soviet actions, he was attempting to convince both the United States public and United States policy makers to adopt a more confrontational position.

The changes in life expectancy in China described in the second paragraph are most likely attributable to which of the following?

By the mid-twentieth century China was still plagued by infectious diseases associated with poverty, while by the end of the twentieth century it had overcome those diseases. China's slow progress in life expectancy in the early twentieth century and fast progress in the late twentieth century described in the passage was most likely attributable to China's ability to confront poverty and the infectious diseases associated with it.

All of the following statements about Canada in the early twentieth century are factually accurate. Which would most likely help to explain why Yanaihara thought that his claims regarding Korea might be well received by a Canadian audience?

Canada's economy grew during most of the early twentieth century under British rule, and Canadian troops willingly helped the British Empire defend its colonial territories in Asia and Africa.Yanaihara may have believed that Canadians' experience of economic growth under British rule and past willingness to defend Great Britain's colonial empire might make them more receptive to his claims that the Japanese occupation of Korea benefited the Korean population and economy.

Which of the following aspects of the immediate historical situation in China in 1958 best explains the author's perspective in the passage?

Chinese leaders had made it clear that meeting the goals of the Great Leap Forward was the highest priority facing the country.Mao Zedong and Chinese leaders implemented a policy program called the Great Leap Forward in 1958. The process involved both mandatory agricultural collectivization and rapid industrialization to usher in a new communist society in China. Chinese leadership set production goals that local party leaders were responsible for achieving. Since the punishments for not achieving these goals were likely to be harsh, party officials such as Yin Zeming often used the inflated rhetoric and exaggerated production numbers that are evident in the passage.

All of the following statements are factually accurate. Which might best be used to undermine or modify the author's argument in the second paragraph?

Christians and Jews in the Ottoman Empire faced considerable restrictions on their religious practices, legal rights, and social freedoms. Masters argues that Christians and Jews received autonomy so that the empire might benefit economically from such minority groups, but his argument that non-Muslim subjects received "fairly wide-ranging" freedoms under Ottoman rule is modified by the fact that the restrictions imposed on non-Muslim communities were considerable.

Which of the flowing best explains how the immediate historical situation of the proclamation of Biafran independence from Nigeria informed the author's statements in the third paragraph?

Claiming to uphold the rule of law and oppose government corruption had become the norm for those seeking political power, even for military leaders seeking to challenge elected governments. Lt. Colonel Ojukwu's listing of the Biafran grievances against the Northern-dominated federal government of Nigeria directly demonstrates that even as a military leader seeking to challenge or overturn an elected government, he feels compelled to invoke the rhetoric of upholding the rule of law and embracing the principles of good government.

Which of the following is an implicit argument made by the author in the passage?

Coerced labor systems have allowed a minority of the population of Caribbean colonial societies to reap enormous economic benefits. In the second paragraph, Labat describes the vast wealth that plantation owners in Barbados have accumulated, and in the first paragraph he mentions that plantation owners were always in fear of slave revolts. In Barbados and across the Caribbean, slaves of African descent constituted the large majority of the population.

Which of the following could best be used to counter Vandana Shiva's criticism of the effects of globalization on the world's agricultural practices, as outlined in the passage?

Data on population and food production trends in developing countries before the advent of industrial farming in the mid-twentieth century. This statement is theoretically correct; however, in practice membership in such organizations is often a prerequisite for developing countries receiving economic aid and sometimes even food aid from the international community. Therefore, the statement does not point to a weakness in Shiva's criticism of global food markets.

Which of the following does the author cite most directly to support his argument in the third paragraph about the importance of exotic goods in the Maya region?

Demarest argues that the exchange of exotic (or "high-status") goods in the Maya region led to the development of unified patterns of behavior in religion, science, and warfare.

Which of the following was a continuity in the development of African states in the period circa 1200-1450 ?

Despite the barrier of the Sahara desert, African states such as Ethiopia and the states of the Sahel (Ghana, Mali, Songhai, etc.) were able to maintain regular diplomatic and cultural contacts with states in North Africa, Mediterranean Europe, and Muslim Southwest Asia.

Which of the following best describes the author's claim in the second paragraph?

Despite the small size of Barbadian plantations, the landowners on Barbados are very wealthy.. The second paragraph states that the plantations on Barbados are "smaller" than those on other Caribbean islands, but it also states that "nonetheless, the plantation owners are very wealthy."

The sixteenth- and seventeenth-century maritime exploration efforts by England, France, and the Netherlands were most directly connected to which of the following historical processes?

Economic competition against Spain and Portugal and a desire to find new sailing routes to Asia. Early English, Dutch, and French exploration and colonization efforts in the sixteenth and the seventeenth centuries were directly motivated by a desire to match and counter the growing economic prosperity and political influence of Spain and Portugal that arose from these countries' imperial expansion.

Which of the following continuities in the development of African states in the period circa 1200-1450 most likely explains the prevalence of Christianity in Ethiopia?

Ethiopia provides a good example of a cultural continuity—Christianity—being used as the basis for state building in Africa. Christianity became widespread among the Ethiopian population and was adopted by Ethiopian political elites as early as the fourth century c.e. Similar examples of a cultural continuity influencing the development of African states would be the influence of Islam in West African states such as Ghana and Mali and in the Swahili states of East Africa in the period 1200-1450. Islam had been introduced to those regions centuries earlier as well. Some African states' religious traditions continued to be influenced by cultural transfers dating back to earlier centuries

Which of the following pieces of evidence most strongly supports the author's conclusion about the importance of exotic goods to the Maya region's economy?

Exotic goods were exchanged over long distances within the Maya lowlands.Demarest claims that exotic goods such as jaguar pelts, textiles, and feathers were exchanged over long distances within the Maya lowlands, thus illustrating the importance of exotic goods to the economy of the wider Maya region.

Which of the following does the author of Source 1 cite as evidence of cultural exchange between East and West Africa during the First World War?

Experiments in architecture and the gathering of souvenirs. Matthews cites experiments in architecture and the gathering of souvenirs as evidence that cultural exchange occurred between East and West African peoples during the First World War. In the first paragraph, Matthews claims that the Hausa, a West African people, used bamboo and grass in their architecture while in East Africa and also collected East African shells and cowries.

The "Hutu Ten Commandments" is best explained as being part of which of the following continuities in twentieth-century history?

Extremist groups using propaganda to target specific minority populations. The "Hutu Ten Commandments" expresses an openly hostile attitude toward an ethnic minority and hints that the majority group ought to take steps to limit the rights of the minority group and, ultimately, use violence against the minority group. This pattern of hostility led directly to the Rwandan genocide and other examples of mass state violence against civilians in the twentieth century, such as the Holocaust and the atrocities against Armenians in the Ottoman Empire.

Which of the following best describes the expansion in total railway line kilometers by 1850 in countries that possessed zero kilometers of railroad lines in 1830 ?

Germany had the largest expansion.Germany had the second-greatest growth in kilometers while also possessing zero kilometers of railroad line in 1830.

Which of the following does the author of Source 1 NOT cite as evidence of European science and technology contributing to cultural change in Nigeria?

Gift gathering for women back home. While Matthews states in the first paragraph that West African troops from Nigeria did gather souvenirs and other collectibles to create necklaces and waist belts for women back home, he does not cite this development as evidence of European science and technology leading to cultural change in Nigeria. Rather, Matthews cites it as evidence of cultural exchange between East and West Africans. Since this option does not provide evidence of how European science and technology contributed to cultural change in Nigeria, it is the correct answer to this question.

As expressed in the passage, Havel's vision of effecting change in the communist bloc was most consistent with which of the following developments?

Grassroots citizens' organizations forming to protest the economic and social injustices in the Communist bloc, such as the Solidarity labor union in Poland. Havel writes about change beginning with dissent at the level of individual thinking, which could then lead to public protest or a social movement. Grassroots citizens' organizations such as the Solidarity labor movement in Poland were examples of how dissent from individuals could provoke collective action that would form into a social movement to protest injustices and eventually lead to political change.

The political activism embodied in the petition is best seen in the context of which of the following broader twentieth-century developments?

Groups and individuals using universal principles to challenge established social norms. The petition represents a group of citizens appealing to the universal principle of equality before the law to challenge established patriarchal social norms and argue that women should be given the vote "on equal terms with men."

Which of the following best explains Lt. Colonel Ojukwu's purpose in including the information in the first paragraph in his independence proclamation?

He wanted to outline a list of East Nigerian grievances in order to justify the political action he was about to take. Lt. Colonel Ojukwu wanted to outline Eastern Nigerian grievances in order to justify rebelling against the Nigerian government to fight for the independence of Eastern Nigeria.

Which of the following best explains why Schama uses the claim that the British government did not have to apply any "undue pressure" on either factory owners or factory workers during the war?

He wanted to support his argument that Britons were completely mobilized for the war effort.He wanted to support his argument that Britons were completely mobilized for the war effort.

Which of the following best explains Niebuhr's sense of alarm about Soviet intentions in occupied Eastern Europe?

He was concerned that Americans would be reluctant to see their recent allies in the Second World War as a threat.Because the very costly Second World War had only just ended in September 1945 and the United States had been an ally of the Soviet Union during the war, Niebuhr feared that Americans would be both unwilling to risk a second conflict and see their former allies as a threat.

Which of the following pieces of information that Trollope reveals about himself in the passage might best explain his contention that economic inequalities ought to be addressed "gradually and without any sudden disruption of society"?

His admission that he was born into a fortunate condition. In the passage, Trollope states that those like him who have been "born in a more fortunate condition" should be "working toward reducing inequalities" in a gradual way that does not erode societal stability and order. It is reasonable to assume that Trollope feared that a sudden or revolutionary change might undermine his own privileged position in society.

Which of the following best describes the historical circumstances of Rashid al-Din's career?

His career illustrates the possibilities for diffusion and intermingling of cultures brought about by the Mongol conquests. Rashid al-Din's position as a Persian at the court of a Mongol ruler and the fact that he oversaw the composition of a universal history illustrates how the Mongol conquests facilitated the transfer of cultural and intellectual traditions.

Which of the following is an argument in the second paragraph of the inscription regarding King Adityawarman?

His spiritual attributes and worldly achievements made him a powerful and legitimate ruler.

The author invokes which of the following to support his claim that smallpox's impact on European settlers was different from its impact on Native Americans?

Ideas about the role of divine providence in human affairs. Bradford credits divine providence, in other words the belief that God is protecting the English settlers who tried to care for the Native Americans stricken by smallpox.

Which of the following developments can be most directly attributed to Portuguese and Spanish state sponsorship of maritime exploration in the late fifteenth century?

Improvements in European geographic knowledge of Africa and the Indian Ocean.Spanish and Portuguese state-sponsored maritime voyages of exploration in the late fifteenth century resulted in the discovery of Atlantic islands, the coast of West and Central Africa, and the discovery of a maritime route to India by rounding the Cape of Good Hope. These discoveries resulted in the improvement of European geographic knowledge of Africa and the Indian Ocean.

A comparison between the data for the three manors shown in the table best illustrates which of the following about the bubonic plague?

In some but not all areas, the population was beginning to recover by 1400.The table shows that in both Great Waltham and High Easter manors, but not in Hatfield Broadoak, the population was beginning to recover by 1400.

The combination between religious and political aspects in the Abbasid state's concept of rulership is best reflected in which of the following?

In the Abbasid caliphate, the caliph theoretically held supreme religious and political authority.

Wilhelm II's assurances in the second paragraph that "nobody is threatening the honor or power of Russia" and that the tsar can "well afford" to await the results of attempts to defuse the crisis best testify to the importance of which of the following factors in the outbreak of the First World War?

Intense nationalism, fueled by mass media, often forced the hand of military and political leaders.Wilhelm II's contention that Russian honor and power were not threatened and that Russia could afford to await the results of his mediation—with the implication that Russia was not threatened militarily by Austria-Hungary or Germany—are a direct appeal to the tsar not to react to nationalist forces inside Russia that demanded war against Austria-Hungary in defense of Serbia and its Orthodox Christian population. Because mass media quickly spread ideas and gave citizens the power to influence political affairs through public criticism, political and military leaders often felt pressure to act. Indeed, both the tsar and his military leaders did not believe that a war against Germany was a good idea but were compelled to act in order to satisfy nationalists in Russia.

Which of the following best describes the change in the total population of males age 12 and older in the three manors shown in the table from 1320 to 1400 ?

It declined to less than a half of its 1320 level (over 50% decline).The population of males age 12 and older in the three manors declined from roughly 1,020 to 496 persons, a decline of slightly more than 50%.

Which of the following best explains how this photograph from the Second World War can be connected to the Cold War that began shortly afterward?

It shows the technological advances made by one of the victors of the Second World War that solidified its position as a postwar superpower. The image shows the assembly of hundreds of powerful B-24 fighter bombers that were used against Japan and Germany during the Second World War. The development of powerful military weapons such as the B-24 and the fact that the United States could produce hundreds of them at a time explain why the United States was able to emerge as a superpower following the Second World War.

Together with access to coal deposits, which environmental factor most directly contributed to Great Britain's early industrialization?

Its abundance of iron ore. Great Britain had plentiful deposits of iron ore, which were critical for creating the industrial machinery in factories.

The changes reflected in which of the following lines in the table are most directly associated with the emergence of debates about the nature and causes of climate change in the late twentieth and early twenty-first century?

Lines 4 and 9. The increases in carbon dioxide emissions (line 4) and deforestation (line 9) have both been directly associated with the emergence of debates over the extent and causes of climate change. Carbon dioxide is a major greenhouse gas and its increase in the atmosphere is considered a major climate change risk. Forests absorb carbon dioxide through photosynthesis, and the decrease in the extent of the world's forests exacerbates the climate change effects of carbon dioxide emissions.

The changes reflected in which of the following lines in the table are most directly associated with international conflicts over scarce natural resources in the late twentieth century?

Lines 5 and 6. There have been numerous international conflicts and disputes in the late twentieth century over division of fishing rights and access to scarce water (usually river water) resources. Examples include the so-called Cod Wars between Iceland and Great Britain; numerous fisheries' conflicts in the South China Sea; conflicts over the building of dams on the Nile, Tigris, and Euphrates rivers; conflicts between post-Soviet Central Asian states over rivers flowing into the Aral Sea, etc.

As outlined in the passage, the provisions of the agreement can best be used to illustrate which of the following aspects of state building in Eurasia in the period circa 1450-1750 ?

Local elites successfully resisted attempts at state centralization and asserted their autonomy from central rule. The agreement clearly demonstrates that Cossack leadership was able to maintain local autonomy in choosing their own leaders and maintaining their own customs.

The pattern of trade described in the passage was most characteristic of which of the following types of goods in the period 1200-1450 ?

Luxury goods such as spices or porcelain. The karim merchants were primarily involved in the trade in luxury goods. Indeed, the use of convoys to protect merchant ships and the establishment of large firms in multiple places along the Indian Ocean trade networks were expensive endeavors for states and merchants that would only be profitable if the goods traded could be sold at high prices. Only the trade in luxury goods such as silks, spices, cottons, and aromatic woods would bring in high enough returns to be worth such large investments.

Which of the following aspects of the Marxist critique of capitalism would be most useful in understanding why the author of the passage chose to invoke Karl Marx's thought in the first paragraph?

Many Marxists criticized imperialism for arresting the economic development of the colonies and reducing them to mere raw material export economies, and Wesseling appears to agree with that criticism.Marx's statement about the "millocracy" that ruled India is consistent with the Marxist criticism that imperialism arrested the economic development of colonies. Wesseling's description of the devastation of the Indian textile industry in the passage and his decision to quote Marx indicate that he agrees with that Marxist criticism.

Which of the following arguments could be most effectively used to undermine Vandana Shiva's implicit criticism of free trade and globalization in the passage?

Many developing countries could never become self-sufficient food producers because of limitations of size, climate, and available cropland. Shiva's criticism of global food markets and the commodification of agriculture implies that she is an advocate of countries pursuing agricultural self-sufficiency. However, food self-sufficiency is not a viable option for many countries around the world—for example, small island countries in the Pacific, or many countries in Africa—irrespective of whether they adopt intensive Green Revolution farming techniques or revert to natural or organic farming methods. Such countries would always have to rely on global food markets to feed their people.

Which of the following would most likely be cited to explain a weakness in Soros' arguments in the passage about the power granted to free-market forces in the global economy?

Market forces are constrained to a large extent by regulations imposed by governments, international economic institutions, and regional trade agreements. Someone could argue that governments still impose significant regulations in free-market economies, that international economic institutions such as the IMF and the World Bank impose some restraints on financial firms and help soften the worst effects of financial and economic crises, and that regional trade agreements also impose a host of regulations and other policies that prevent market forces from obtaining "free rein" as Soros argues in the passage.

The rapid growth of industrial output in Russia during the 1890s was most likely comparable to the growth of industrial output in which other state during the same period?

Meiji Japan. Meiji Japan industrialized its economy by studying and modeling Western economies and through close cooperation between the Meiji government and private conglomerate companies called zaibatsus. These efforts allowed Japan to become an industrial power by the end of the nineteenth century.

As portrayed by Ho Chi Minh in the passage, the Vietnamese independence movement appears most similar to which of the following types of nineteenth-century and early-twentieth-century movements?

Nationalist movements. In the passage, Ho Chi Minh uses the ideas of national sacrifice and liberation and saving the "Fatherland" to motivate the Vietnamese people to resist the French. These ideas are most similar to those used by nineteenth- and twentieth-century nationalists.

Which of the following is an accurate comparison of the positions expressed by the authors of the two sources regarding the idea that existing political boundaries should be changed so that each nation has its own state?

Neither Source 1 nor Source 2 explicitly supports the idea. Neither Anthimos nor Bessenyei explicitly supports the idea of redrawing political boundaries to grant each nation its own state. Anthimos's argument is based on the implicit belief that imperial boundaries protect the interests of religious minorities. Bessenyei, while expressing nationalist sentiments, does not explicitly advocate changing existing boundaries.

Which of the following would best support Schama's argument in the first paragraph about the role of Churchill's speeches in Great Britain's war effort?

New mass media such as radio greatly expanded the reach of governments' mobilization efforts.Radio and other mass media made it possible for governments during the Second World War to reach the entirety of their nation and others living abroad, inspiring patriotism and mobilization efforts. Britain, in particular, utilized mass communication to create a unified population dedicated to a common goal: the defeat of the Germans.

Based on the graph, which of the following best describes the development of transportation and communication in the twentieth century?

New methods of transportation and communication became more affordable over time. The costs of all four technologies in the graph declined rapidly over the twentieth century. This development was most likely due to economies of scale and improvements in the technologies and their manufacture and delivery over time.

Which of the following best explains why people from nonindustrialized regions constituted the majority of migrants in the nineteenth and early twentieth centuries?

Nonindustrialized regions often experienced large population increases that made it more difficult for people to find local jobs. Because of agricultural advances such as better methods of crop rotation, more intensive use of fertilizer, or the ongoing spread of new crops like the potato, some nonindustrialized regions, such as southern Italy, Ireland, India, and China, experienced large increases in their total populations, which made it difficult for people to find work locally and encouraged many to emigrate.

Which of the following is the main claim that the author makes in the passage?

People have natural rights that can be established through rational inquiry.At the beginning of the first paragraph, Condorcet states that "philosophers have at last discovered the true rights of man" and these rights can be "deduced from the single truth: that man is a perceptive being capable of reason and acquiring moral ideas." This shows that Condorcet is arguing that people's natural rights have been determined through rational philosophical inquiry. The idea of natural rights is also developed in the third paragraph of the passage.

The policy position of the Manitoba Liberal Party quoted in the appeal is best seen as a continuation of which of the following broader trends in the late 1800s and early 1900s?

Political parties embracing mass politics and broader popular participation in the democratic process. The expansion of voting rights (for men) and increased access to education in countries such as Canada led to the transformation of party politics, with the rise of new political parties and with existing parties having to adapt their programs to reflect the views of a widening electorate.

Which of the following best describes the historical situation in Eurasia at the time the passage was written?

Political upheaval and reorganization increased cultural and scientific exchanges across regions.The establishment of the Mongol Empire and the subsequent expansion of the Ilkhanate led to the destruction of numerous Islamic states. Despite the destruction that these conquests caused, the Mongol expansion also facilitated cultural and scientific exchanges as illustrated by the fact that Muslim, Christian, and Chinese astronomers worked together at Maragha and that the Mongols ordered the construction of observatories in their realm.

The contents of the letter are best understood in the context of which of the following aspects of the historical situation in sub-Saharan Africa in the early 1600s?

Political, religious, and economic rivalries shaped European colonial policies in Africa. Just as European rivalries often contributed to European imperialistic endeavors in the Americas and Asia, in Africa competition between European states shaped European expansion into the region and provided opportunities for African leaders to play European powers against one another. The letter reflects this competition between European states in its criticism of Portuguese policies and its praise for the king of Congo's actions against the Portuguese.

Which of the following is an implicit claim that the author makes in the second paragraph?

Religion has been a force holding back the progress of humanity.While Condorcet does not specifically mention religion in the second paragraph, he does state that people could now use reason to search for truth and no longer had to base their beliefs on the "superstitions of antiquity." These statements indicate that he believes that religion had been holding back human progress.

Witbooi's perspective in the last paragraph is historically significant because it most directly illustrates which of the following rationales for African people resisting European imperial encroachment?

Resistance to preserve traditional political structures threatened by direct European territorial expansion. Witbooi's leadership of this revolt against German authority represented an effort to maintain his family's leadership of one group of the Nama people as they sought to carve out a secure territorial homeland outside the reach of European control.

The passage can best be used to explain the role of which of the following in bringing about the First World War?

Rival great power alliances. The passage shows that Russia was mobilizing its forces against Austria-Hungary and that Wilhelm II of Germany would go to war with Russia if it attacked Germany's ally, Austria-Hungary. Such competing alliance systems among the great powers of Europe were directly responsible for bringing about the First World War.

The fact that the Russian tsar insisted in the last paragraph that some ambassadors be treated differently from others best illustrates which of the following about international politics in the period circa 1650-1750 ?

Russia had encountered significant resistance to its plans for further territorial expansion from some of the established states in eastern and southern Europe.The tsar's concerns about ambassadors from powerful established states in Europe indicates that Russia likely experienced difficulties expanding into those areas.

Which of the following events that preceded the outbreak of the First World War best explains why Russian troops were being mobilized against Austria-Hungary, as stated in the first paragraph?

Russia was acting in support of Serbia, which was facing an imminent Austro-Hungarian attack following the assassination of Archduke Franz Ferdinand.After the assassination of the Archduke Franz Ferdinand by a Serbian nationalist, Austria-Hungary threatened Serbia with war, and Russia was mobilizing its forces to protect Serbia because it was a Russian ally and a fellow Eastern Orthodox nation.

he makeup of the force shown in the image is best explained by the fact that by 1917

Russia's army morale and home-front effort were collapsing because of the strains of total war. Russia's ability to mobilize its populace to sacrifice for the war effort was greatly challenged by the fact that the army had lost a series of significant battles and experienced heavy casualties. As a result, discontent both among soldiers and workers—especially workers in factories producing munitions and other materials related to the war effort—was very high. Eventually, this discontent led to the uprising against the Tsarist regime whose immediate aftermath is depicted in the image.

Based on the graph, which technology was developed most recently?

Satellite television. Based on the graph, satellite television was developed in the mid 1970s.

Which of the following best explains the significance of the intended audience of Yin Zeming's pamphlet in shaping the author's narrative?

Since it targets an international audience across the Cold War divide, the pamphlet presents an idealized, propaganda-tinged vision of Chinese accomplishments. The fact that the document makes overt political statements, celebrates Chinese accomplishments with no mention of the negative aspects of party policies, and was published in English by a foreign-language press indicates that the text was likely intended to influence public opinion in Western states during the Cold War.

The image and its caption provide evidence that would best help explain the ideas of which of the following ideologies that emerged in the late nineteenth century?

Social Darwinism. Social Darwinism applied Darwinian theories of natural selection to justify racial categorization of human populations similar to those seen in the caption of the social studies textbook. These categories served to justify racism and imperial expansion during the nineteenth and twentieth centuries.

Trollope's arguments in the second paragraph are most clearly intended to offer an alternative to the arguments made by which late nineteenth-century group?

Socialists. Late nineteenth-century socialists aimed to radically expand the electorate, distribute the profits of business more equally, and provide greater financial and legal protections to the poor through higher taxes on the wealthy, the adoption of public welfare programs, and greater government regulation of business. Some socialists even argued that the government should seize businesses and establish state monopolies in order to achieve greater social and economic equality. Because the adoption of socialist policies would involve a radical departure from existing political and social structures in late nineteenth-century Britain, Trollope's arguments in the second paragraph about gradual social change are intended to offer an alternative to socialist policies. Indeed, Trollope directly argues in the second paragraph that the adoption of communism (a radical form of socialism) would bring ruin.

Which of the following best explains how a supporter of free trade and market deregulation would counter Soros' critiques of free markets in the passage?

Soros ignores the fact that the adoption of free-market policies by states around the world in the late twentieth century increased standards of living for many people. A supporter of free trade and market deregulation would argue that Soros' emphasis on the problems of free-market economic policies ignores the greater good that these policies produce in lifting standards of living.

Which of the following is an accurate comparison between the views expressed in the two sources?

Source 1 emphasizes the importance of religious identities and solidarities as drivers of human behavior, whereas Source 2 emphasizes the importance of ethno-linguistic identities and solidarities as drivers of human behavior.Source 1 emphasizes that true liberty is a Christian liberty, namely the freedom to live according to both the divine laws of God and the human laws of the Church and the Ottoman state. Source 2 emphasizes the importance of language and culture in motivating human behavior, arguing that language and birth were the most significant factors in defining identity and that Hungarians should strive for glory.

Which of the following is an accurate comparison of the arguments made in the two sources regarding ideas of liberty and equality?

Source 1 rejects the ideas because of the threat they represent to multinational empires, whereas Source 2 rejects the ideas because of the universal and equalizing nature of their claims.Source 1 rejects the ideas because of the threat they represent to multinational empires, whereas Source 2 rejects the ideas because of the universal and equalizing nature of their claims.

The trends in military personnel and state revenue shown in the table are best understood in the context of which of the following changes in the relationship between states and local elites in the period 1450-1750 ?

States increasingly centralized their authority at the expense of local elites. While local elites still retained significant power, states' abilities to maintain standing armies and acquire larger and larger amounts of revenue allowed states to centralize their authority and reduce the power of local elites.

Which of the following best describes a claim made in the first paragraph of the inscription?

Statues of Buddhist divine figures could spiritually benefit everyone. The first paragraph of the inscription states that a statue of a bodhisattva was dedicated "for the benefit and salvation and happiness of all creatures."

Which of the following is an implicit claim that the author makes in the third paragraph?

The French Revolution has advanced the principles of Enlightenment.Condorcet wrote during the early stages of the French Revolution. While he does not explicitly describe the French Revolution in the third paragraph, it is the clear context for this passage. In the third paragraph, he claims that the various Enlightenment principles and policies that he mentions have filtered down through every level of society and have become the common faith of all people.

The rival British and French claims over the Niger River region in the 1890s were most directly a part of which of the following processes?

The European "scramble" to acquire territories in Africa through both peaceful and military means. The letter from the directors of the British Royal Niger Company illustrates British attempts to secure its influence in West Africa and limit French influence through peaceful means preferably, but military means if necessary. British-French competition in West Africa in the late nineteenth century took place in the context of the "Scramble for Africa," during which European powers attempted to secure their respective spheres of influence in Africa by claiming colonial territories.

The spread of Islam into sub-Saharan West Africa in the period circa 1200-1450 was mostly a result of the conversion of the rulers of which of the following states?

The Mali Empire. The rulers of the Mali Empire converted to Islam under the influence of Berber traders from North Africa in the early thirteenth century. Other African political leaders, such as those of the sultanate of Kano in the grasslands regions that bordered the Sahara, soon followed suit.

The confrontation illustrated in the two photographs can be seen as a specific example of the global rivalries in the period after 1945 that also directly produced which of the following?

The NATO and Warsaw Pact military alliances. The NATO and Warsaw Pact military alliances formed directly in response to Cold War tensions between anticommunist states in Western Europe that were supported by the United States and communist satellite states in Eastern Europe supported by the Soviet Union. NATO formed as a result of United States attempts to protect Western Europe from a potential Soviet attack, while the Warsaw Pact in Eastern Europe formed under Soviet leadership to counter the threat posed by NATO.

Which of the following aspects of the international situation in early sixteenth-century southwest Asia is most relevant to understanding Sultan Selim's letter?

The Ottoman and Safavid empires fought numerous wars for control over Iraq, Syria, and other regions of the Middle East. The Safavids and the Sunni Ottoman Turks fought several wars over control of various parts of the Middle East during the sixteenth century, and each side employed rhetoric and propaganda characterizing the other as religious heretics.

Which of the following would a historian most likely cite as evidence in support of the author's argument in the second paragraph of the passage?

The Ottoman millet system, an arrangement designed to utilize the economic contributions of non-Muslim groups within the empire while granting them limited autonomy to organize their communal affairs under their own religious leaders. The millet system supports Masters's argument that the Ottoman Empire was willing to extend limited autonomy to minority groups for its financial benefit.

Japanese expansion in the late nineteenth century is most directly explained in the context of the decline of which of the following empires?

The Qing Empire. The context of the photograph is the First Sino-Japanese War (July 1894-April 1895) in which an industrializing Japan inflicted a humiliating defeat on the Qing Empire. The resulting loss of Korea, parts of China, Taiwan, and other territories, as well as the imposition of costly reparation payments, were seen as bitter evidence of Qing China's weakened world position.

Which of the following developments in the late twentieth century would most likely be cited to explain a flaw in Soros' arguments regarding "social needs" and "collective decision making" in the first paragraph?

The Soviet Union and other communist states in Eastern Europe collapsed largely because of economic inefficiencies created by centralized economic planning and attempts to eliminate all economic and social inequalities.Centralized economic planning in the name of supposed attempts to provide economic equality for everyone created enormous inefficiencies in communist economies, which led to popular discontent and the collapse of communist regimes in Eastern Europe and the Soviet Union. This development could be cited to demonstrate a flaw in Soros' argument that market forces should be curbed to address greater social needs.

Which of the following aspects of the immediate historical situation in which the letter was written best explains the author's demands toward the end of the second paragraph?

The Soviet government used the pretext of the ongoing civil war to engage in repressive policies against entire sections of its population.The author's demands in the second paragraph that the bourgeoisie be destroyed and that its property be redistributed to workers can be explained in the context of what eventually became known as the Red Terror, during which Bolshevik forces engaged in a widespread campaign of mass executions, imprisonments, forced military mobilization, and property confiscations against anyone perceived as supporting the former Tsarist regime, including many peasants and even some workers.

The unrest reflected in the image is best explained by which of the following features of the Russian government at the time of the photograph?

The Tsarist regime that governed Russia was autocratic and unresponsive to calls for reform.During the late-nineteenth and early-twentieth centuries, civil unrest broke out on many occasions against the Tsarist government because of its repressive policies and corruption and because of the plight of industrial workers. Most of these instances of unrest were caused by radical socialist and communist groups. The tsar's regime responded by using force and the secret police to intimidate its opponents and by implementing piecemeal reforms that did not go nearly far enough to satisfy its opponents.

All of the following statements about the Hutu and Tutsi ethnic groups in Rwanda are factually accurate. Which would best explain how the attitudes expressed in the "Hutu Ten Commandments" were a reaction to past developments in Rwandan history?

The Tutsi were heavily favored and promoted to positions of power under European colonial rule, while the Hutu were not.European colonial authorities tended to favor the Tutsi over the Hutu because they perceived the Tutsi to be more "racially" advanced and more "civilized" than the Hutu. Hutu resentment over this unjust treatment under colonial rule lingered postindependence and directly informed interethnic tensions in independent Rwanda, as shown in the "Hutu Ten Commandments."

In addition to Japan, which of the following non-European states created an empire in the Asia-Pacific region in the late nineteenth century?

The United States. Following the American Civil War, the United States established a presence in the Asia-Pacific region, opening up Japan to American trade in 1853 with the expedition of Commodore Matthew Perry and later acquiring coaling stations and naval outposts throughout the region before acquiring the Philippines in the aftermath of the Spanish-American War of 1898.

Despite the disintegration of the Abbasid Caliphate, Islam continued to spread across Afro-Eurasia in the period 1200-1450 primarily because of which of the following?

The activities of Sufi missionaries were the largest factor in the further spread of Islam across Afro-Eurasia in the period 1200-1450, as Sufis won new converts in places such as Anatolia, Central Asia, India, and Southeast Asia where Islam had already been introduced.

In the period circa 1200-1450, which of the following most directly enabled merchants to bring salt and other bulk products to markets in sub-Saharan Africa?

The adoption of innovative practices in overland trading, such as the use of camel caravans and saddles. The Muslim conquest of North Africa in the eighth century encouraged an increase in trans-Saharan trade, largely carried by camel caravans, some with thousands of camels. Such large caravans made possible an expansion of trade in bulk items such as salt.

A historian studying the collapse of the Soviet Union in the early 1990s would most likely use the letter above to illustrate which of the following?

The alienation of Soviet citizens from their government's official rhetoric of class struggle and anti-capitalism. As the letter shows, in private correspondence Soviet citizens showed no interest in the official government ideology of class struggle and anti-capitalism. On the contrary, they were quite willing to embrace the products of the capitalist world and the consumer tastes of their Western counterparts.

The passage implies that the author was aware of smallpox being an infectious disease. Which of the following can best be cited as evidence of that claim?

The author's statement that no English settlers fell ill, even though "many performed these favors for the Indians for weeks". This statement shows that Bradford recognized there was risk in caring for Native Americans afflicted by smallpox because he expresses relief and gratitude that no English settlers fell ill.

Which of the following does the author of Source 2 most directly cite as evidence of a higher casualty rate among Senegalese soldiers in combat than among French soldiers in combat?

The casualty rate after July 1916. In the second paragraph, Lunn notes that in the last two and a half years of the First World War, beginning with casualty numbers in 1916, Senegalese combat losses were "approximately twice as high as those suffered by French infantry combatants." Therefore, Lunn cites the casualty rate after July of 1916 as the most direct evidence that the Senegalese casualty rate was higher than the French casualty rate.

Which of the following contributed directly to Great Britain's ability to consolidate or expand its colonial control of interior regions of Asia and Africa in the late nineteenth century?

The changes illustrated in the top two rows of the image. Railroads and steam-powered vessels were critical factors in allowing British troops, officials, and companies to access interior regions of Africa and Asia that had been previously inaccessible and consolidate their power.

The authors' discussion in the third paragraph of "the marked acceleration in health care improvements" in China after 1950 refers most directly to which of the following historical developments?

The communist victory in China's civil war. The communists' victory in 1949 and their ability to establish a stable and powerful state was the key transition in the mid-century that could explain the healthcare advances described in the passage.

The image is best understood in which of the following contexts of both the Second World War and the Cold War?

The continued importance of industry in improving military capacity. The image of B-24 bombers being assembled in a plant in Texas directly illustrates how industry was critical in boosting military capacity during both the Second World War and, later, the Cold War. Indeed, during the Cold War not only did the military-industrial complex of both the United States and the Soviet Union produce large quantities of military equipment that made both states superpowers, but industry also helped both to develop more technologically sophisticated military equipment than other states possessed.

Which of the following is best supported by the information shown on the graph?

The cost of technologies that rely on the use of petroleum declined less rapidly than the cost of technologies that do not rely on the use of petroleum.The graph demonstrates that, of the four technologies shown, the decline in the cost of the two that rely on the use of petroleum (ocean freight and air transport) was less rapid than the decline in the cost of the two technologies that do not rely on the use of petroleum (trans-Atlantic telephone calls and satellite television).

Considering global trends in late-nineteenth-century migration movements, the Indian migrations to the Caribbean that produced the demographic situation shown in the table most likely led to which of the following short-term effects?

The creation of Indian ethnic enclaves in Caribbean societies. The vast majority of indentured workers from India initially lived in ethnic enclaves closely tied to plantation zones.

Which of the following developments in the period 1450-1750 is best understood as a response to the trends shown in the table?

The creation of larger state bureaucracies. Because states developed standing armies and had more revenue coming in to their respective treasuries, they needed more bureaucrats to manage the larger armies and revenues.

The economic changes in rapidly industrializing countries such as the Russian Empire in the late nineteenth century led most directly to which of the following social or intellectual changes during the same period?

The creation of new social classes and class consciousness. Rapid industrialization in the nineteenth century often led to the creation or enlargement of new social classes, such as the industrial working class, and the development of class consciousness, which often led to political and economic agitation.

The terminology used in the opening sentence of the decree was most directly influenced by which of the following?

The deep influence of Christianity on medieval society and culture. Phrases such as "by the grace of God" and "reverend father in Christ" reflect the deep religiosity of medieval society and political culture.

Which of the following best describes the author's argument in the first paragraph?

The demographic makeup of Barbadian society and the structure of its economy make existing punishments for slaves justified. . Labat argues in the first paragraph that White plantation owners in Barbados are justified in punishing their slaves harshly because slaves are necessary for the economic well-being of the island and because slaves constitute the majority of the inhabitants.

The debates about maritime shipping in Japan alluded to in the first paragraph were most directly connected to which of the following nineteenth-century developments?

The development of rival economic theories differing with respect to their views of free trade. Iwasaki's allusion to debates within Japan about whether allowing foreign companies to participate in Japan's maritime trade would be beneficial or harmful to Japan is most directly connected to arguments between those who argued that allowing foreign competition would increase economic competition and thereby ultimately benefit Japanese companies by making them more efficient and those—like Iwasaki—who argued that protecting Japanese companies from foreign competition was necessary because exposing Japanese companies to foreign competition would lead to a loss of jobs for Japanese citizens and endanger Japan's security.

Which of the following best explains why transoceanic migration increased significantly in the nineteenth and early twentieth centuries?

The development of steam-powered ships made it easier and safer for people to cross the oceans.The development of steam engines allowed for the creation of larger, faster, and safer vessels that could carry more people to their destinations more quickly while reducing the risk of vessels sinking because of bad weather and large waves.

Wesseling's argument in the second paragraph best illustrates which of the following important distinctions within the practice of imperialism in the nineteenth century?

The difference between the economic effects of direct and indirect imperialism. Wesseling argues that because India was ruled by Great Britain directly (whether through the English East India Company or by the Crown), it could not erect tariff barriers to protect its textile industry. In Latin America and other parts of Asia, however, Western states practiced indirect economic imperialism. Although Latin American and Asian states that experienced Western economic imperialism often had a very difficult time protecting their industries, they could do so; their local industries often partnered with Western companies, and they often did not experience the same degree of economic devastation as in directly ruled colonies such as India.

The disintegration of the Abbasid Caliphate most directly led to which of the following political developments in the Islamic world in the thirteenth century?

The disintegration of the Abbasid state resulted in the emergence of numerous Islamic states controlled by Turkic peoples. Long before the collapse of the Abbasid Caliphate in 1258, the Islamic world split into a number of smaller successor states, most of which were dominated by Turkic ruling classes. In the thirteenth century, some examples of such states included the Delhi Sultanate of India and the Mamluk Sultanate of Egypt.

Based on the pattern of trade described in the third paragraph, the Egyptian karim merchants were most likely directly involved in which of the following broader developments in the Indian Ocean in the period circa 1200-1450 ?

The establishment of diasporic merchant communities. The pattern of trade described in the passage most directly illustrates that the karim merchants held property across the Indian Ocean trading networks, which more likely indicates the establishment of diasporic merchant communities. Indeed, the karim merchants often maintained branches in both Egypt and along the coast of India, where members of the firm settled in local ports to organize business transactions, acquire products to be shipped to Egypt, and ensure that the goods brought from Egypt would be sold in India.

Considering the purpose of the letter, it most likely overstates which of the following?

The extent to which wealthy Russians continued to live exactly as they did before the revolution (second paragraph).Because the purpose of the letter is to claim that the government should seize all private property still in the hands of wealthy Russians, its author is most likely overstating the extent to which the bourgeoisie continues to live surrounded by the material comforts they enjoyed before the Bolshevik Revolution. The author of the letter likely used such overstatement to help him achieve his goal of convincing the government to implement his demands.

Which of the following best summarizes the way in which the development of the factory system and the development of new transportation infrastructure such as railways worked together as factors facilitating British industrialization?

The factory system concentrated production in relatively few locations, and the new transportation infrastructure allowed more goods and people to reach these locations in less time.In eighteenth- and nineteenth-century Britain, industrial production was primarily concentrated in cities such as London, Manchester, and Liverpool, and the development of new transportation technologies such as railways facilitated the migration of people to cities to work in factories and allowed factories to acquire supplies from and send their products to other markets in Britain.

The process of "consultations with the nobles and clergy of the realm" in the making of Edward III's decree best reflects which of the following?

The fragmented, decentralized nature of European feudal monarchies. Like most European states at the time, the English monarchy had to contend with the power of an entrenched hereditary nobility and a semiautonomous clerical establishment. While the monarch's power was absolute in theory, in practice he or she often needed to wield it in cooperation or consultation with the aristocracy and the clergy.

The letter above could best be used to illustrate which of the following aspects of the historical context of the late twentieth century?

The globalization of consumer culture despite the rivalries of the Cold War. The letter provides direct evidence that Western popular and consumer culture had a strong influence among Soviet youth, despite state censorship and Cold War rivalries.

Which other process in the mid to late nineteenth century most directly led to situations in which non-Western countries found it very difficult to protect their indigenous industries from Western competition, as described in the passage?

The imposition of free-trade regimes on countries such as China, the Ottoman Empire, and Latin American countries through military or diplomatic pressure from Western countries. In the mid to late nineteenth century, Western states used military and diplomatic pressure in Latin America, the Ottoman Empire, and China to impose free-trade regimes that greatly benefited Western companies because free trade allowed Western companies to dump their goods into Latin American and Asian markets at lower prices than local companies.

The rapid growth of nineteenth-century industrial cities such as Manchester is best understood in the context of which of the following economic developments?

The increases in agricultural productivity that freed up laborers for work in factories. The development of new agricultural and transportation technologies and more efficient use of land allowed for greater crop yields that could feed more people while simultaneously requiring less labor in the agricultural sector. Unemployed or underemployed farm laborers had to find jobs in industrial factories such as Manchester, which led to the rapid growth of those towns.

Which of the following best describes a difference between the purpose of the institution established in Source 1 and the purpose of the institutions established in Source 2 ?

The institution established in Source 1 was intended to prevent war, while the institutions established in Source 2 were intended to promote economic development and international trade.The first agreement states that the United Nations and the Security Council were intended to maintain peace through collective action, while the second agreement states that the goals of the World Bank and the IMF were to promote economic development—especially in states devastated by war—and expand international trade.

The type of warfare described in the passage most directly helps to explain which of the following sixteenth-century developments?

The intensification of political rivalries between the Ottoman Empire and neighboring states in Europe and the Middle East. Gunpowder weapons helped increase the substantial military power of Ottoman armies and allowed them to conquer most of southern Europe and much of the Middle East in the sixteenth century. The Ottomans' growing power alarmed European powers and the Safavid Empire in Iran during the sixteenth century, both of which sought to build their military arsenals to resist the Ottomans.

Trollope's characterization of democracy in the second paragraph can best be seen as a direct reference to which aspect of the historical situation in the late nineteenth century?

The movements to reform countries' political systems by expanding the franchise. Political reform movements in nineteenth-century European states had their roots in Enlightenment thought and, therefore, often advocated for the rapid expansion of the franchise. Demands for the expansion of the franchise frequently led to violence and political unrest, such as in Great Britain during the Chartist movement and across Europe during the revolutions of 1848.

Which of the following best describes a main idea common to both passages?

The new organizations should facilitate greater international cooperation. Both agreements state that the newly established organizations are intended to facilitate greater international cooperation to address a number of international problems.

All of the following statements are factually accurate. Which would best support the author's argument in the first paragraph of the passage?

The period of Mamluk rule in Egypt saw a high number of conversions to Islam and a considerable reduction in the percentage of Coptic Christians in the overall population of Egypt.Masters argues that Christianity in the Middle East was in psychological and numerical decline, which would be supported by the fact that Coptic Christian populations in Egypt declined because of conversions to Islam.

Which of the following developments or processes in the late twentieth century best explains the participation of foreign troops in the Angolan Civil War, as shown in the two photographs?

The proliferation of Cold War proxy conflicts. The two photographs show soldiers from communist Cuba and the Western-aligned South African Republic participating in the Angolan Civil War. To avoid direct conflict that could lead to a nuclear war, the United States and the Soviet Union used their respective allies, in what are referred to as proxy conflicts, to help fulfill their ideological and political objectives.

The point of view that Hitler expresses in the first three paragraphs is most directly significant in helping to explain how which of the following contributed to starting the Second World War?

The provisions of the Treaty of Versailles that ended the First World War. The Treaty of Versailles transferred German colonial holdings to other European and Asian powers, which deeply angered German nationalists and contributed significantly to Germany's desire to reacquire its former territories. Germans' sense of grievance at the harsh provisions contained in the Treaty of Versailles was exploited by the Nazis to acquire the popular support necessary for engaging in policies that began the Second World War. In the passage, Hitler states that the German colonies were "taken away from us unreasonably" under the terms of the Treaty of Versailles.

The economic statistics cited by Wesseling in the first paragraph are most likely intended to illustrate which aspect of the nineteenth-century global context?

The role of British imperial policy in reversing the development of Indian textile manufacturing. The statistics that Wesseling cites demonstrate that the Indian textile industry was devastated by the ability of British textile mills to mass-produce cloth on a scale with which Indian textile workers could not compete.

The general pattern of nomadic conquest followed by the assimilation of the conquerors into the cultures of the conquered societies was most clearly expressed in which of the following developments in the period circa 1250-1450 ?

The rulers of the Yuan dynasty adopting Chinese court culture and methods of rule. The Mongol conquerors of China established the Yuan dynasty and slowly adopted Chinese court rituals and bureaucratic methods of governance necessary to control their Chinese territories

The image could best be understood as evidence for which of the following developments?

The shift in the global balance of power associated with the Second World War. The ability of the United States to mass produce military weapons during the Second World War was a major reason for the shift in the global balance of power following the war's conclusion because the economic devastation experienced by Western European states such as France and Great Britain guaranteed that those states would not be able to compete militarily with the United States or the Soviet Union.

Which of the following developments in the period after 1945 most directly prevented localized conflicts such as the one illustrated in the two photographs from escalating into global wars?

The threat of the use of nuclear weapons. The potential for mutually assured destruction through the use of nuclear weapons prevented direct conflict between the United States and the Soviet Union during the Cold War. Hence, although the United States and the Soviet Union often placed considerable economic and military resources into localized proxy wars like the Angolan Civil War and saw those conflicts as essential to achieving their respective geopolitical and ideological objectives, they avoided direct conflict with each other.

As described in the passage, the pattern of economic development in Russia in the nineteenth century differed most strongly from which other pattern of economic development during the same period?

The transition from preindustrial to industrial production through the actions of private entrepreneurs or companies. The pattern of transitioning from a preindustrial to an industrial economy through private entrepreneurs and investment, as was the typical pattern in the West, was most different from the pattern of industrialization in Russia, which was largely dictated, managed, and financed by the state.

The conduct of the siege as described in the passage could best be used to explain which of the following processes in the period 1450-1750 ?

The use of new weapons by expanding imperial states. The Ottoman Empire was an expanding imperial state and was using gunpowder weapons to capture Constantinople and destroy the Byzantine Empire. Gunpowder weapons were only used in warfare on a large scale beginning in the fourteenth century.

The passage can best be used to illustrate which of the following developments in the 1970s and 1980s?

The way in which government efforts to suppress criticism and restrict freedom of speech proved ineffective in stemming public discontent within communist regimes. Communist regimes in Eastern Europe did attempt to stifle critiques and free speech, even with military crackdowns such as that in Hungary in 1956 and in Czechoslovakia following the Prague Spring in 1968. However, they were never able to stop all dissent, and by the 1970s and 1980s more people within communist states were expressing discontent publicly.

Which of the following best explains the long-term historical significance of the views expressed by Yanaihara in the passage?

They show that some members of the Japanese elite sought to justify aggressive Japanese colonial efforts in China and the Pacific region.In 1938, Japan had already embarked upon a war with China and would soon extend its colonial reach across the Pacific, threatening United States and European colonial powers' interests in the region. Yanaihara's letter shows that Japanese academics and intellectuals attempted to defend and justify this expansion to Western audiences by using Western notions of the civilizing mission and economic progress.

Which of the following best characterizes the significance of the events in Russia in the winter of 1917, as shown in the image?

They started a process that eventually led to a communist revolution.The February Revolution in Russia overthrew the monarchy and established a provisional government led by liberal and socialist parties. However, the ongoing pressure of the war and the inability of the provisional government to check the spread of communist agitation in Russian cities eventually led to the so-called October Revolution in which Lenin's Bolshevik party seized power and established a communist government.

Some African states' religious traditions continued to be influenced by cultural transfers dating back to earlier centuries

They used religion and laws to bolster their legitimacy. Like virtually all rulers throughout history, African rulers invoked both religious claims and their role as lawgivers or law enforcers to bolster their political legitimacy. For example, the ambassadors' description of King Zara Yacob indicates that he sought to project and advertise his power in these terms as well.

Which of the following most likely explains why Niebuhr chose to publish his article in a popular publication like Life magazine?

To change government policies in a democratic country like the United States, it was first necessary to change public opinion.Niebuhr understood that in a democratic country like the United States, swaying public opinion often places pressure on elected officials for policy change. Life magazine, as a popular magazine aimed at a broad and general audience, offered a platform to reach millions of Americans.

Which of the following best describes the author's purpose in the passage?

To portray his patron as a generous supporter of the sciences and a man of unusual intelligence. Rashid al-Din claims that Ghazan Khan ordered the construction of a dome at the Maragha observatory and the construction of an observatory at Tabriz, that he understood the complex astronomical instruments that he studied at Maragha, that he explained how to construct a complex observatory at Tabriz to local wise men, and that those wise men marveled at Ghazan Khan's intelligence.

Which of the following best explains why the author brings up the example of the Sikh migrant cutting his hair before coming to England as a way of supporting his argument that the experiences of South Asian and European immigrants to Britain were "poles apart"?

Unlike European immigrants, South Asian immigrants had to suppress parts of their culture and remove important religious symbols to fit into British society.Wills argues that, unlike European immigrants, South Asian immigrants discarded parts of their cultural and religious heritage, such as the Sikh migrant's long hair—a crucial part of Sikh religious identity—to better assimilate into British society and to improve their chances of securing jobs.

Which of the following best explains a difference between European and South Asian migration during the nineteenth and early twentieth centuries?

Unlike European migrants, South Asian migrants often served as indentured laborers. Although Irish, Scottish, and English migrants to Australia were convict laborers, European migrants rarely became indentured servants. South Asians, however, frequently migrated under indentured servant contracts to Africa and the Caribbean.

As indicated by the passage, Vietnam's achievement of independence differed in what way from the achievement of independence by many other former colonies?

Vietnam gained its independence through a violent uprising rather than a negotiated settlement. The passage illustrates that the Vietnamese used violence to win their independence, unlike some former European colonies, such as India.

Which aspect of the historical situation in West Africa in the 1890s best explains the board of directors' instructions to Lugard in article (h) of the letter?

West Africa was becoming a source of important raw materials for Europe's industrializing economies.Article (h) directs Lugard to record the occurrence of gum arabic trees, shea butter trees, and rubber vines in the regions he is about to visit. All three of these trees were sources of prized raw materials for European industries such as food processing, soap making, and tire production. The singling out of these particular plant species suggests that West Africa was seen by the members of the board of directors as a potential source of industrial raw materials for European factories.

Iwasaki's position in the letter on the relationship between the Mitsubishi Company and the Japanese government is most clearly a response to

Western governments gaining access to Japan's markets by imposing laissez-faire economic regimes. In the third paragraph, Iwasaki claims that the Mitsubishi company needs government assistance to be able to compete with Western companies such as the British Peninsular and Oriental Steam Navigation Company, which Iwasaki clearly saw as a threat. Iwasaki's concern about the threat of the P&O Company to his business is directly related to his remarks in the second paragraph that Japan had been forced to sign treaties with foreign powers that granted them access to Japanese ports and allowed them to transport Japanese goods to other regions, which is a reference to the treaties that the Tokugawa government was forced to sign by the United States and other Western states that imposed free-trade regimes on Japan.

The commercial rivalries discussed in the third paragraph best illustrate the interplay between nineteenth-century economic development in Japan and the development of

Western-dominated large-scale transnational businesses. The British Peninsular and Oriental Steam Navigation Company that Iwasaki refers to in his letter was a transnational business that operated across the world and was seeking to acquire greater dominance in East Asia. These companies began emerging in the nineteenth century, primarily with the aim of acquiring natural resources from Asia, Africa, and Latin America for industrial factories in Western states.

Which of the following aspects of the international context in which the petition was produced most likely strengthened the petitioners' demands?

Women's critical contributions to the home front effort during The First World War. The mobilization of the Allied powers during the First World War put millions of women to work in positions in medicine, manufacturing, and other fields that broadened the advocacy for the woman suffrage movement.

Which of the following best explains a likely significant purpose behind Yanaihara publishing his article in a Canadian journal?

Yanaihara wished to justify continued Japanese colonial rule of Korea to potentially hostile Western states.Western powers justified continued rule of their own colonies with many of the same arguments employed by Yanaihara in this editorial, and Yanaihara's use of those same arguments in a Canadian journal indicates that he intended to win acceptance of Japan's policies in Western states that may have been alarmed at Japanese expansion.

Considering the global context of the 1970s, which of the following would have been the most significant difference between young people living outside the Communist bloc and the two young men in the source?

Young people outside the Communist bloc would have been unlikely to need to buy Western goods on the black market. The records and clothes discussed in the letter were readily available for purchase in the open markets of most noncommunist countries.

Waterways such as canals and rivers were important in the process of early industrialization because they provided

a means for integrating economic activities in regional and national markets. Easy access to rivers and canals in Great Britain provided the ability for industrial factories to acquire supplies from and ship their products to regional and national markets, which helped lower production costs and increase profits.

Based on the purpose of the letter, Selim's description of the followers of Shah Ismail I as a "heretical faction" can best be interpreted as a commentary of the Safavid Empire's

adherence to Shi'a Islam. The Safavid rulers adopted Shi'a Islam, which set them apart from the Sunni rulers of the Ottoman Empire. This religious rivalry would be the best explanation for Selim's use of the derogatory term "heretical" to describe the Safavid followers.

The disparities of wealth mentioned by Parkinson in the passage are best explained in the context of the

high profits and low wages associated with manufacturing industries in the nineteenth century. The high profits acquired by the rich and low wages paid to the industrial workers led to the disparities of wealth between the social classes.

The claim in the second paragraph that the king has become "free from all physical desire" can best be understood to mean that

in Vedic religions, rejecting worldly concerns in order to attain spiritual perfection was considered a virtue. Being free of physical desires in order to concentrate on spiritual pursuits is a key belief of Vedic religions such as Buddhism and Hinduism. The fact that the passage refers to the king as being free from all physical desire in the context of praising him for dedicating a statute to a bodhisattva and of being "an ocean of all virtues practiced by the followers of the Buddha" indicates that the phrase is used in this religious sense of the phrase.

In the context of the late 1930s, the arguments that Hitler makes in the last paragraph are most significant in explaining how

imperialist aspirations contributed to motivating the policies of fascist states.In the last paragraph, Hitler argues that the difficult economic circumstances that Germany faced and Germany's high population density required that it obtain colonies, implying that Germany should either regain the colonies that it once possessed before the end of the First World War or acquire new colonies. The fact that the speech was made in 1937, when Germany was already actively pursuing an aggressive and expansionist policy in Europe, shows that a desire to regain Germany's colonies was also an important driving force behind fascist state policies.

Witbooi's point of view in discussing his right to rule the Nama lands "by double conquest" is historically significant mostly because it illustrates how

interethnic conflicts in Africa led to political fragmentation and ultimately limited Africans' ability to resist imperialism. Interethnic rivalries like those described in Witbooi's letter frequently provided European leaders with the opportunity to divide and conquer African communities.

The privileges discussed in the last two paragraphs of the passage best demonstrate that in some instances in the period 1450-1750

internal challenges to state power could lead to the establishment of new state-like structures within existing states. The tsar's acceptance of Cossack requests in the final two paragraphs indicates a substate formation within the larger Russian Empire. The Cossacks were required to do certain things, like collect taxes for the tsar, but they were also granted the responsibility to administer their own affairs on a local level.

In the late twentieth century, members of conservationist and environmentalist movements would most likely have cited the data in all of the following lines of the table as support for their concerns EXCEPT

lines 1 and 2. Members of conservationist and environmentalist movements might have indirectly used the growth of the global economy to illustrate their concerns (for example, to illustrate the unsustainability of growth), but they would have been much more likely to cite the direct environmental impacts expressed in the other lines in the table.

Witbooi's statement in the last paragraph that he would likely be killed in the process of resisting the Germans is historically significant because it shows that Africans understood the

low probability that armed resistance would be successful because of the military advantages of Europeans. Witbooi was under no illusions about the ability of the small population of Nama people to withstand advanced German firepower and was aware that most African attempts to resist Europeans by force had failed.

Based on the author's point of view, it is most likely that the information presented in the passage

overstates the extent to which Northern Nigerians were responsible for undermining the united Nigerian state. Because Lt. Colonel Ojukwu was advocating for the use of force to separate Eastern Nigeria from the rest of the country, he most likely presented information that overstated the extent to which Northern Nigerians were responsible for undermining the united Nigerian state.

The author directly uses all of the following pieces of evidence to support his argument about the relationship between exotic goods and the power of Maya rulers EXCEPT that Maya rulers

prohibited nobles and priests from wearing certain types of exotic goods reserved for rulers

Based on the context of European imperialism in the late nineteenth century, a historian examining the image and its caption would likely explain that they provide evidence of how France sought to

reassure students that France's inhumane colonial policies were justified by a so-called "civilizing mission". The racial categories identified in the caption sustained European notions of the innate superiority of people of European descent and consequent right to rule. These categories offered justification for policies that were designed to "better" or "civilize" colonial societies and cultures.

Based on the historical situation in which the speech was given, Hitler's most significant motivation for giving the speech was likely to continue to

reinforce the idea to the German people that past injustices required Germany to continue its military buildup and engage in future territorial expansion. Hitler's speech emphasizes the wrongs of the Treaty of Versailles, which robbed Germany of its colonial holdings. Given that the speech was issued to the German Parliament in 1937 before the Second World War, Hitler was likely trying to remind the German people of this perceived injustice and justify Germany's aggressive military buildup and future territorial expansion.

The argument in the third paragraph regarding the connection between the Partition of India in 1947 and Punjabi migrations to Britain in the 1950s is most directly supported by the author's claim that

resettling Partition refugees led to an economic crisis in rural Punjab, leading to further population movements. Wills argues that South Asian migrants principally came to Britain from the Punjab because of the economic crisis caused by the Partition of India, which forced rural Punjabis to attempt to survive on much smaller parcels of land.

The report's position on the proposed transfer of Luanda to the Dutch is best understood in light of the authors' goal to

secure profits from the rapidly expanding trans-Atlantic slave trade. The report's mention of the fact that Luanda is "so useful to the King of Spain" because of the large numbers of slaves shipped from there to the Americas indicates that the Company's directors are seeking to convince the Dutch government that they should be more involved in the growing (and very profitable) trans-Atlantic slave trade.

The point of view expressed by the Dutch West India Company's directors in the letter can best be described as

seeking to steer the States General representatives into taking action that would benefit the company financially. The letter specifically states that the Dutch West India Company stood to gain financially by acquiring control over forts on the West African coast from which the company could trade for slaves from the Angola region that would be vital to the establishment of lucrative sugar-growing plantations in the Caribbean.

Compared to the position of anticolonial movements earlier in the twentieth century, the prospects for movements such as Ho Chi Minh's in the mid-twentieth century could most accurately be described as

stronger because the Second World War weakened imperialist powers. Although Great Britain and France were among the victorious Allied powers during the Second World War, both states suffered considerable economic devastation, and their military power was greatly weakened. Hence, anticolonial movements in colonies such as Vietnam and Algeria were successful in the mid-twentieth century, whereas they would likely have failed in the early twentieth century.

The letter's significance as a source of information on popular attitudes to Bolshevik revolutionary policies lies in the fact that it suggests that some Soviet citizens

supported the Bolsheviks in their nationalization of industrial property but urged them to seize the upper classes' private property as well. The author argues in the passage that even though the Bolsheviks had nationalized industrial factories and the businesses of the bourgeoisie, the bourgeoisie was still enjoying fairly luxurious lifestyles and, therefore, their property and possessions should be confiscated and distributed to the workers.

The need for legal action to address the "grave inconveniences" that might result from the labor shortages described in the passage most likely resulted from

the agrarian nature of medieval European economies and their reliance on coerced labor. The sense of urgency behind the labor shortages of agricultural workers, particularly ploughmen, reflects the agrarian nature of medieval economies, and the mention of serfs reflects the reliance of medieval economies on coerced labor.

The authors' argument in the third paragraph concerning the effect of "social factors" on the effectiveness of a country's healthcare system likely refers to all of the following general trends in the twentieth century EXCEPT

the high likelihood that increasing longevity would lead to an increase in overall healthcare costs. Increasing healthcare costs would not likely contribute to improved healthcare outcomes. This incorrect statement is the exception, making it the correct response.

The Mongol conquests resulted in all of the following developments EXCEPT

the initial diffusion of Buddhism and Christianity to East Asia. Buddhism and Christianity had spread into East Asia well before the Mongol conquests. While the Mongols expanded cultural exchanges across Eurasia, the particular religious transfers mentioned in the answer choice do not represent a result of the Mongol conquests. Therefore, this option is the correct answer to the question.

In the period circa 1750-1900, the historical process illustrated by the image led directly to all of the following major global transformations EXCEPT

the outbreak of the Atlantic revolutions and the emergence of new states in the Americas. Although the Industrial Revolution began in Great Britain in the mid-eighteenth century, it was not a factor that led to the Atlantic revolutions and the emergence of new states in the Americas. Since this option does not illustrate a direct connection between the process illustrated in the image and a major global transformation in the period 1750-1900, it is the correct answer.

Contrary to the author's implicit argument in the second paragraph regarding South Asian migrants' determination to integrate into British society, the migration of former colonial subjects to imperial metropoles typically resulted in

the preservation of cultural and economic ties between former colonies and metropoles, as immigrant communities maintained some connections to their countries of origin. Despite Wills's argument that most South Asian migrants quickly abandon their cultural traditions in order to assimilate into British society, most immigrants—including most South Asian migrants to Britain—tried to maintain their cultural heritage and economic ties with their country of origin and usually maintained connections to their home countries, sometimes by sending money to family in the home country or by visiting their home countries on occasion.

In addition to the factors exemplified by the passage, the end of the Cold War was also directly hastened by all of the following EXCEPT

the signing of nuclear arms control treaties between the United States and the Soviet Union. Although the United States and the Soviet Union did sign nuclear arms control treaties in the 1970s and the 1980s, these treaties did not directly lead to the end of the Cold War. Since this option represents a factor that did not lead to the end of the Cold War, it is the correct answer to this question.

Unlike the four scenes in the top two rows of the lithograph, the inclusion of the two scenes in the bottom row is best explained by

the technological innovations of the second industrial revolution. The top two rows show the transition to steamships and railways from sailing vessels and animal-powered transportation as part of the First Industrial Revolution, while the bottom row shows electric lighting in major cities, which was a product of the second industrial revolution. The First Industrial Revolution, which lasted from the mid-eighteenth century to the mid-nineteenth century, was characterized by new forms of transportation like steam power and railways. The spread of electricity, telecommunications, improvements made to steel manufacturing, and the invention of interchangeable parts all characterized the second industrial revolution of circa 1870-1914.

The integration of West African states into wider regional and transregional economic networks in the period circa 1200-1450 was carried out mostly via the

trans-Saharan trade routes. The trans-Saharan trade routes extended from the coast of the Mediterranean across the Sahara Desert and connected West African states such as the Hausa Sultanate, the Ghana Empire, and the Mali Empire to wider regional and transregional economic networks.

A historian would most likely interpret the rhetoric in the passage as evidence that rulers of imperial states in the period circa 1450-1750 continued to

use religious ideas to highlight their political legitimacy and attack the legitimacy of their enemies. Sultan Selim's rhetoric in the passage presents a sharp contrast between the idealized and honorific terms in which Selim refers to himself and his realm and the disparaging and belittling terms that Selim uses to describe his political enemy, the shah of Safavid Persia.

The ideas behind the establishment of the council referred to in Source 1 and the ideas behind the establishment of the bank referred to in Source 2 are most similar in that

while the council and the bank were intended to provide benefits to all members, a small group of states were allowed to retain permanent decision-making power. The first agreement states that five member states of the Security Council would have a permanent group of members who could make decisions regarding maintaining peace, while the second agreement states that the World Bank would have five permanent executive directors from its five largest shareholders.


Ensembles d'études connexes

Mike Meyers A+ Certification Flash Cards

View Set

Developing an Innovation Strategy

View Set

Chapter 11: Building Nursing Management Skills Zerwekh 9th ED

View Set

rev management exam 2 quiz chapter 4

View Set

Mendel's Laws Applied to Complex Traits

View Set